Matrices similarity in a bigger field $K$ Implies matrices similarity in the smaller field $F$. The 2019 Stack Overflow Developer Survey Results Are In Announcing the arrival of Valued Associate #679: Cesar Manara Planned maintenance scheduled April 17/18, 2019 at 00:00UTC (8:00pm US/Eastern)Similar matrices and field extensionsSimilarity of Matriceswhy similarity over $barmathbbF$ of $A,Bin M_n(mathbbF)$ implies similarity over $mathbbF$?2 matrices veryfing similaritySimultaneous similarity of pairs of matricesfinite classes of similarity relation on $n times n$ matrices over $M_n(mathbbF)$Similarity classes of matricesOn the similarity form a matrix with $A^m=E$relation between two positive definite matrices via similarity transformation.Similarity of matrices based on polynomialsField extension similarity

How did the audience guess the pentatonic scale in Bobby McFerrin's presentation?

Can the prologue be the backstory of your main character?

What is special about square numbers here?

Why can't devices on different VLANs, but on the same subnet, communicate?

Wall plug outlet change

How to split my screen on my Macbook Air?

Sort a list of pairs representing an acyclic, partial automorphism

Single author papers against my advisor's will?

Would an alien lifeform be able to achieve space travel if lacking in vision?

How is simplicity better than precision and clarity in prose?

How can I define good in a religion that claims no moral authority?

What LEGO pieces have "real-world" functionality?

Why is superheterodyning better than direct conversion?

Semisimplicity of the category of coherent sheaves?

University's motivation for having tenure-track positions

Didn't get enough time to take a Coding Test - what to do now?

Are my PIs rude or am I just being too sensitive?

What are these Gizmos at Izaña Atmospheric Research Center in Spain?

Python - Fishing Simulator

Is it ethical to upload a automatically generated paper to a non peer-reviewed site as part of a larger research?

Can a novice safely splice in wire to lengthen 5V charging cable?

ELI5: Why do they say that Israel would have been the fourth country to land a spacecraft on the Moon and why do they call it low cost?

Do warforged have souls?

Derivation tree not rendering



Matrices similarity in a bigger field $K$ Implies matrices similarity in the smaller field $F$.



The 2019 Stack Overflow Developer Survey Results Are In
Announcing the arrival of Valued Associate #679: Cesar Manara
Planned maintenance scheduled April 17/18, 2019 at 00:00UTC (8:00pm US/Eastern)Similar matrices and field extensionsSimilarity of Matriceswhy similarity over $barmathbbF$ of $A,Bin M_n(mathbbF)$ implies similarity over $mathbbF$?2 matrices veryfing similaritySimultaneous similarity of pairs of matricesfinite classes of similarity relation on $n times n$ matrices over $M_n(mathbbF)$Similarity classes of matricesOn the similarity form a matrix with $A^m=E$relation between two positive definite matrices via similarity transformation.Similarity of matrices based on polynomialsField extension similarity










3












$begingroup$


I have a Linear Algebra exercise and I have trouble solving a part of it.
The follwing question shows us that if $K subseteq L$ is a field extension such that both $L,K$ are infinite ($L,K$ are fields) and $A,B in M_n(K)$ such that $A,B$ are similar in the field $L$ (or: there exists an invertible matrix $P in M_n(L)$ such that $PA=BP$) then $A,B$ are already similar in the field $K$ (or: there exists an invertible matrix $P in M_n(K)$ such that $PA=BP$). I need to prove that this way:



(1). Show that every non-zero polynomial $f in K[x_1,...,x_n]$ there exists $lambda_1,...,lambda_n in K$ such that $f(lambda_1,...,lambda_n)neq 0$. Do that using induction and show that this is necessary that $K$ is infinite (find a counter example for finite $K$)


(2). Suppose $f in K[x_1,...,x_n]$ is a polynomial such that there are $lambda_1,...,lambda_n in L$ such that $f(lambda_1,...,lambda_n) neq 0$.
Show that there are $mu_1,...,mu_n in K $ such that $f(mu_1,...,mu_k) neq 0$.



(3) Assume that there exists invertible $P in M_n(L)$ such that $PA=BP$.
Show that there exists scalars $a_1,...,a_r in L$ and matrices $P_1,...,P_r in M_n(K)$ such that the set $a_1,...,a_r$ is $K$-linearly independent and also $a_1P_1+...+a_rP_r = P$. Show that $P_iA = BP_i$ for all $i$.

(4) Show that there exists $b_1,...,b_r in K$ such that $b_1P_1+...+b_rP_r$ is invertible (Hint: use (2) with $f(x_1,...,x_r) = det(x_1P_1+...+x_rP_r)$

(5) Conclude from (4) and (3) that there exists an invertible matrix $Q in M_n(K)$ such that $QA=BQ$.



I was able to solve everything but part (3). I tried searching that in google and all I found was this: Similar matrices and field extensions
And there he just uses part (3) as guaranteed. How do I prove that?










share|cite|improve this question











$endgroup$
















    3












    $begingroup$


    I have a Linear Algebra exercise and I have trouble solving a part of it.
    The follwing question shows us that if $K subseteq L$ is a field extension such that both $L,K$ are infinite ($L,K$ are fields) and $A,B in M_n(K)$ such that $A,B$ are similar in the field $L$ (or: there exists an invertible matrix $P in M_n(L)$ such that $PA=BP$) then $A,B$ are already similar in the field $K$ (or: there exists an invertible matrix $P in M_n(K)$ such that $PA=BP$). I need to prove that this way:



    (1). Show that every non-zero polynomial $f in K[x_1,...,x_n]$ there exists $lambda_1,...,lambda_n in K$ such that $f(lambda_1,...,lambda_n)neq 0$. Do that using induction and show that this is necessary that $K$ is infinite (find a counter example for finite $K$)


    (2). Suppose $f in K[x_1,...,x_n]$ is a polynomial such that there are $lambda_1,...,lambda_n in L$ such that $f(lambda_1,...,lambda_n) neq 0$.
    Show that there are $mu_1,...,mu_n in K $ such that $f(mu_1,...,mu_k) neq 0$.



    (3) Assume that there exists invertible $P in M_n(L)$ such that $PA=BP$.
    Show that there exists scalars $a_1,...,a_r in L$ and matrices $P_1,...,P_r in M_n(K)$ such that the set $a_1,...,a_r$ is $K$-linearly independent and also $a_1P_1+...+a_rP_r = P$. Show that $P_iA = BP_i$ for all $i$.

    (4) Show that there exists $b_1,...,b_r in K$ such that $b_1P_1+...+b_rP_r$ is invertible (Hint: use (2) with $f(x_1,...,x_r) = det(x_1P_1+...+x_rP_r)$

    (5) Conclude from (4) and (3) that there exists an invertible matrix $Q in M_n(K)$ such that $QA=BQ$.



    I was able to solve everything but part (3). I tried searching that in google and all I found was this: Similar matrices and field extensions
    And there he just uses part (3) as guaranteed. How do I prove that?










    share|cite|improve this question











    $endgroup$














      3












      3








      3


      1



      $begingroup$


      I have a Linear Algebra exercise and I have trouble solving a part of it.
      The follwing question shows us that if $K subseteq L$ is a field extension such that both $L,K$ are infinite ($L,K$ are fields) and $A,B in M_n(K)$ such that $A,B$ are similar in the field $L$ (or: there exists an invertible matrix $P in M_n(L)$ such that $PA=BP$) then $A,B$ are already similar in the field $K$ (or: there exists an invertible matrix $P in M_n(K)$ such that $PA=BP$). I need to prove that this way:



      (1). Show that every non-zero polynomial $f in K[x_1,...,x_n]$ there exists $lambda_1,...,lambda_n in K$ such that $f(lambda_1,...,lambda_n)neq 0$. Do that using induction and show that this is necessary that $K$ is infinite (find a counter example for finite $K$)


      (2). Suppose $f in K[x_1,...,x_n]$ is a polynomial such that there are $lambda_1,...,lambda_n in L$ such that $f(lambda_1,...,lambda_n) neq 0$.
      Show that there are $mu_1,...,mu_n in K $ such that $f(mu_1,...,mu_k) neq 0$.



      (3) Assume that there exists invertible $P in M_n(L)$ such that $PA=BP$.
      Show that there exists scalars $a_1,...,a_r in L$ and matrices $P_1,...,P_r in M_n(K)$ such that the set $a_1,...,a_r$ is $K$-linearly independent and also $a_1P_1+...+a_rP_r = P$. Show that $P_iA = BP_i$ for all $i$.

      (4) Show that there exists $b_1,...,b_r in K$ such that $b_1P_1+...+b_rP_r$ is invertible (Hint: use (2) with $f(x_1,...,x_r) = det(x_1P_1+...+x_rP_r)$

      (5) Conclude from (4) and (3) that there exists an invertible matrix $Q in M_n(K)$ such that $QA=BQ$.



      I was able to solve everything but part (3). I tried searching that in google and all I found was this: Similar matrices and field extensions
      And there he just uses part (3) as guaranteed. How do I prove that?










      share|cite|improve this question











      $endgroup$




      I have a Linear Algebra exercise and I have trouble solving a part of it.
      The follwing question shows us that if $K subseteq L$ is a field extension such that both $L,K$ are infinite ($L,K$ are fields) and $A,B in M_n(K)$ such that $A,B$ are similar in the field $L$ (or: there exists an invertible matrix $P in M_n(L)$ such that $PA=BP$) then $A,B$ are already similar in the field $K$ (or: there exists an invertible matrix $P in M_n(K)$ such that $PA=BP$). I need to prove that this way:



      (1). Show that every non-zero polynomial $f in K[x_1,...,x_n]$ there exists $lambda_1,...,lambda_n in K$ such that $f(lambda_1,...,lambda_n)neq 0$. Do that using induction and show that this is necessary that $K$ is infinite (find a counter example for finite $K$)


      (2). Suppose $f in K[x_1,...,x_n]$ is a polynomial such that there are $lambda_1,...,lambda_n in L$ such that $f(lambda_1,...,lambda_n) neq 0$.
      Show that there are $mu_1,...,mu_n in K $ such that $f(mu_1,...,mu_k) neq 0$.



      (3) Assume that there exists invertible $P in M_n(L)$ such that $PA=BP$.
      Show that there exists scalars $a_1,...,a_r in L$ and matrices $P_1,...,P_r in M_n(K)$ such that the set $a_1,...,a_r$ is $K$-linearly independent and also $a_1P_1+...+a_rP_r = P$. Show that $P_iA = BP_i$ for all $i$.

      (4) Show that there exists $b_1,...,b_r in K$ such that $b_1P_1+...+b_rP_r$ is invertible (Hint: use (2) with $f(x_1,...,x_r) = det(x_1P_1+...+x_rP_r)$

      (5) Conclude from (4) and (3) that there exists an invertible matrix $Q in M_n(K)$ such that $QA=BQ$.



      I was able to solve everything but part (3). I tried searching that in google and all I found was this: Similar matrices and field extensions
      And there he just uses part (3) as guaranteed. How do I prove that?







      linear-algebra abstract-algebra matrices extension-field






      share|cite|improve this question















      share|cite|improve this question













      share|cite|improve this question




      share|cite|improve this question








      edited Apr 8 at 14:29







      Omer

















      asked Apr 8 at 13:43









      OmerOmer

      535110




      535110




















          1 Answer
          1






          active

          oldest

          votes


















          2












          $begingroup$

          Consider the finite-dimensional $K$-vector subspace $V$ of $L$ spanned by all the entries of the matrix $P$. Let $a_1, dots, a_r$ be a basis of $V$ over $K$. Then the $(j, k)$ entry of $P$ can be written as (I use exponents for the entries of a matrix, since indices are taken for another role)
          $$
          P^jk
          =
          a_1 P^jk_1 + dots + a_r P^jk_r
          $$

          for suitable $P^jk_i in K$. Now $P_i$ is the matrix whose $(j, k)$ component is $P_i^j k$.



          We have
          $$
          a_1 (P_1 A) + dots + a_r (P_r A)
          =
          P A
          =
          B P
          =
          a_1 (B P_1) + dots + a_r (B P_r).
          $$

          Now consider each component of this matrix identity:
          $$
          a_1 (P_1 A)^jk + dots + a_r (P_r A)^jk
          =
          a_1 (B P_1)^jk + dots + a_r (B P_r)^jk.
          $$

          Since the $a_i$ are independent over $K$, and the $(P_i A)^jk, (B P_i)^jk$ are in $K$, this shows that $(P_i A)^jk = (B P_i)^jk$ for each $i, j, k$, so that $P_i A = B P_i$ for all $i$.






          share|cite|improve this answer











          $endgroup$












          • $begingroup$
            I am very sorry, I made a mistake writing the question. A and B should be matrices over K, and not over L. Your point 3 solution is still working?I think it does, but there is one problem: How do you know that there exists a finite basis of F over K?
            $endgroup$
            – Omer
            Apr 8 at 14:33










          • $begingroup$
            Nevermind, I think it's because there is finite amount of entries of P?
            $endgroup$
            – Omer
            Apr 8 at 14:38










          • $begingroup$
            Just finished/fixed my full answer.
            $endgroup$
            – Andreas Caranti
            Apr 8 at 14:38












          Your Answer








          StackExchange.ready(function()
          var channelOptions =
          tags: "".split(" "),
          id: "69"
          ;
          initTagRenderer("".split(" "), "".split(" "), channelOptions);

          StackExchange.using("externalEditor", function()
          // Have to fire editor after snippets, if snippets enabled
          if (StackExchange.settings.snippets.snippetsEnabled)
          StackExchange.using("snippets", function()
          createEditor();
          );

          else
          createEditor();

          );

          function createEditor()
          StackExchange.prepareEditor(
          heartbeatType: 'answer',
          autoActivateHeartbeat: false,
          convertImagesToLinks: true,
          noModals: true,
          showLowRepImageUploadWarning: true,
          reputationToPostImages: 10,
          bindNavPrevention: true,
          postfix: "",
          imageUploader:
          brandingHtml: "Powered by u003ca class="icon-imgur-white" href="https://imgur.com/"u003eu003c/au003e",
          contentPolicyHtml: "User contributions licensed under u003ca href="https://creativecommons.org/licenses/by-sa/3.0/"u003ecc by-sa 3.0 with attribution requiredu003c/au003e u003ca href="https://stackoverflow.com/legal/content-policy"u003e(content policy)u003c/au003e",
          allowUrls: true
          ,
          noCode: true, onDemand: true,
          discardSelector: ".discard-answer"
          ,immediatelyShowMarkdownHelp:true
          );



          );













          draft saved

          draft discarded


















          StackExchange.ready(
          function ()
          StackExchange.openid.initPostLogin('.new-post-login', 'https%3a%2f%2fmath.stackexchange.com%2fquestions%2f3179647%2fmatrices-similarity-in-a-bigger-field-k-implies-matrices-similarity-in-the-sma%23new-answer', 'question_page');

          );

          Post as a guest















          Required, but never shown

























          1 Answer
          1






          active

          oldest

          votes








          1 Answer
          1






          active

          oldest

          votes









          active

          oldest

          votes






          active

          oldest

          votes









          2












          $begingroup$

          Consider the finite-dimensional $K$-vector subspace $V$ of $L$ spanned by all the entries of the matrix $P$. Let $a_1, dots, a_r$ be a basis of $V$ over $K$. Then the $(j, k)$ entry of $P$ can be written as (I use exponents for the entries of a matrix, since indices are taken for another role)
          $$
          P^jk
          =
          a_1 P^jk_1 + dots + a_r P^jk_r
          $$

          for suitable $P^jk_i in K$. Now $P_i$ is the matrix whose $(j, k)$ component is $P_i^j k$.



          We have
          $$
          a_1 (P_1 A) + dots + a_r (P_r A)
          =
          P A
          =
          B P
          =
          a_1 (B P_1) + dots + a_r (B P_r).
          $$

          Now consider each component of this matrix identity:
          $$
          a_1 (P_1 A)^jk + dots + a_r (P_r A)^jk
          =
          a_1 (B P_1)^jk + dots + a_r (B P_r)^jk.
          $$

          Since the $a_i$ are independent over $K$, and the $(P_i A)^jk, (B P_i)^jk$ are in $K$, this shows that $(P_i A)^jk = (B P_i)^jk$ for each $i, j, k$, so that $P_i A = B P_i$ for all $i$.






          share|cite|improve this answer











          $endgroup$












          • $begingroup$
            I am very sorry, I made a mistake writing the question. A and B should be matrices over K, and not over L. Your point 3 solution is still working?I think it does, but there is one problem: How do you know that there exists a finite basis of F over K?
            $endgroup$
            – Omer
            Apr 8 at 14:33










          • $begingroup$
            Nevermind, I think it's because there is finite amount of entries of P?
            $endgroup$
            – Omer
            Apr 8 at 14:38










          • $begingroup$
            Just finished/fixed my full answer.
            $endgroup$
            – Andreas Caranti
            Apr 8 at 14:38
















          2












          $begingroup$

          Consider the finite-dimensional $K$-vector subspace $V$ of $L$ spanned by all the entries of the matrix $P$. Let $a_1, dots, a_r$ be a basis of $V$ over $K$. Then the $(j, k)$ entry of $P$ can be written as (I use exponents for the entries of a matrix, since indices are taken for another role)
          $$
          P^jk
          =
          a_1 P^jk_1 + dots + a_r P^jk_r
          $$

          for suitable $P^jk_i in K$. Now $P_i$ is the matrix whose $(j, k)$ component is $P_i^j k$.



          We have
          $$
          a_1 (P_1 A) + dots + a_r (P_r A)
          =
          P A
          =
          B P
          =
          a_1 (B P_1) + dots + a_r (B P_r).
          $$

          Now consider each component of this matrix identity:
          $$
          a_1 (P_1 A)^jk + dots + a_r (P_r A)^jk
          =
          a_1 (B P_1)^jk + dots + a_r (B P_r)^jk.
          $$

          Since the $a_i$ are independent over $K$, and the $(P_i A)^jk, (B P_i)^jk$ are in $K$, this shows that $(P_i A)^jk = (B P_i)^jk$ for each $i, j, k$, so that $P_i A = B P_i$ for all $i$.






          share|cite|improve this answer











          $endgroup$












          • $begingroup$
            I am very sorry, I made a mistake writing the question. A and B should be matrices over K, and not over L. Your point 3 solution is still working?I think it does, but there is one problem: How do you know that there exists a finite basis of F over K?
            $endgroup$
            – Omer
            Apr 8 at 14:33










          • $begingroup$
            Nevermind, I think it's because there is finite amount of entries of P?
            $endgroup$
            – Omer
            Apr 8 at 14:38










          • $begingroup$
            Just finished/fixed my full answer.
            $endgroup$
            – Andreas Caranti
            Apr 8 at 14:38














          2












          2








          2





          $begingroup$

          Consider the finite-dimensional $K$-vector subspace $V$ of $L$ spanned by all the entries of the matrix $P$. Let $a_1, dots, a_r$ be a basis of $V$ over $K$. Then the $(j, k)$ entry of $P$ can be written as (I use exponents for the entries of a matrix, since indices are taken for another role)
          $$
          P^jk
          =
          a_1 P^jk_1 + dots + a_r P^jk_r
          $$

          for suitable $P^jk_i in K$. Now $P_i$ is the matrix whose $(j, k)$ component is $P_i^j k$.



          We have
          $$
          a_1 (P_1 A) + dots + a_r (P_r A)
          =
          P A
          =
          B P
          =
          a_1 (B P_1) + dots + a_r (B P_r).
          $$

          Now consider each component of this matrix identity:
          $$
          a_1 (P_1 A)^jk + dots + a_r (P_r A)^jk
          =
          a_1 (B P_1)^jk + dots + a_r (B P_r)^jk.
          $$

          Since the $a_i$ are independent over $K$, and the $(P_i A)^jk, (B P_i)^jk$ are in $K$, this shows that $(P_i A)^jk = (B P_i)^jk$ for each $i, j, k$, so that $P_i A = B P_i$ for all $i$.






          share|cite|improve this answer











          $endgroup$



          Consider the finite-dimensional $K$-vector subspace $V$ of $L$ spanned by all the entries of the matrix $P$. Let $a_1, dots, a_r$ be a basis of $V$ over $K$. Then the $(j, k)$ entry of $P$ can be written as (I use exponents for the entries of a matrix, since indices are taken for another role)
          $$
          P^jk
          =
          a_1 P^jk_1 + dots + a_r P^jk_r
          $$

          for suitable $P^jk_i in K$. Now $P_i$ is the matrix whose $(j, k)$ component is $P_i^j k$.



          We have
          $$
          a_1 (P_1 A) + dots + a_r (P_r A)
          =
          P A
          =
          B P
          =
          a_1 (B P_1) + dots + a_r (B P_r).
          $$

          Now consider each component of this matrix identity:
          $$
          a_1 (P_1 A)^jk + dots + a_r (P_r A)^jk
          =
          a_1 (B P_1)^jk + dots + a_r (B P_r)^jk.
          $$

          Since the $a_i$ are independent over $K$, and the $(P_i A)^jk, (B P_i)^jk$ are in $K$, this shows that $(P_i A)^jk = (B P_i)^jk$ for each $i, j, k$, so that $P_i A = B P_i$ for all $i$.







          share|cite|improve this answer














          share|cite|improve this answer



          share|cite|improve this answer








          edited Apr 8 at 14:40

























          answered Apr 8 at 14:25









          Andreas CarantiAndreas Caranti

          57.2k34497




          57.2k34497











          • $begingroup$
            I am very sorry, I made a mistake writing the question. A and B should be matrices over K, and not over L. Your point 3 solution is still working?I think it does, but there is one problem: How do you know that there exists a finite basis of F over K?
            $endgroup$
            – Omer
            Apr 8 at 14:33










          • $begingroup$
            Nevermind, I think it's because there is finite amount of entries of P?
            $endgroup$
            – Omer
            Apr 8 at 14:38










          • $begingroup$
            Just finished/fixed my full answer.
            $endgroup$
            – Andreas Caranti
            Apr 8 at 14:38

















          • $begingroup$
            I am very sorry, I made a mistake writing the question. A and B should be matrices over K, and not over L. Your point 3 solution is still working?I think it does, but there is one problem: How do you know that there exists a finite basis of F over K?
            $endgroup$
            – Omer
            Apr 8 at 14:33










          • $begingroup$
            Nevermind, I think it's because there is finite amount of entries of P?
            $endgroup$
            – Omer
            Apr 8 at 14:38










          • $begingroup$
            Just finished/fixed my full answer.
            $endgroup$
            – Andreas Caranti
            Apr 8 at 14:38
















          $begingroup$
          I am very sorry, I made a mistake writing the question. A and B should be matrices over K, and not over L. Your point 3 solution is still working?I think it does, but there is one problem: How do you know that there exists a finite basis of F over K?
          $endgroup$
          – Omer
          Apr 8 at 14:33




          $begingroup$
          I am very sorry, I made a mistake writing the question. A and B should be matrices over K, and not over L. Your point 3 solution is still working?I think it does, but there is one problem: How do you know that there exists a finite basis of F over K?
          $endgroup$
          – Omer
          Apr 8 at 14:33












          $begingroup$
          Nevermind, I think it's because there is finite amount of entries of P?
          $endgroup$
          – Omer
          Apr 8 at 14:38




          $begingroup$
          Nevermind, I think it's because there is finite amount of entries of P?
          $endgroup$
          – Omer
          Apr 8 at 14:38












          $begingroup$
          Just finished/fixed my full answer.
          $endgroup$
          – Andreas Caranti
          Apr 8 at 14:38





          $begingroup$
          Just finished/fixed my full answer.
          $endgroup$
          – Andreas Caranti
          Apr 8 at 14:38


















          draft saved

          draft discarded
















































          Thanks for contributing an answer to Mathematics Stack Exchange!


          • Please be sure to answer the question. Provide details and share your research!

          But avoid


          • Asking for help, clarification, or responding to other answers.

          • Making statements based on opinion; back them up with references or personal experience.

          Use MathJax to format equations. MathJax reference.


          To learn more, see our tips on writing great answers.




          draft saved


          draft discarded














          StackExchange.ready(
          function ()
          StackExchange.openid.initPostLogin('.new-post-login', 'https%3a%2f%2fmath.stackexchange.com%2fquestions%2f3179647%2fmatrices-similarity-in-a-bigger-field-k-implies-matrices-similarity-in-the-sma%23new-answer', 'question_page');

          );

          Post as a guest















          Required, but never shown





















































          Required, but never shown














          Required, but never shown












          Required, but never shown







          Required, but never shown

































          Required, but never shown














          Required, but never shown












          Required, but never shown







          Required, but never shown







          Popular posts from this blog

          Hidroelektrana Sadržaj Povijest | Podjela hidroelektrana | Snaga dobivena u hidroelektranama | Dijelovi hidroelektrane | Uloga hidroelektrana u suvremenom svijetu | Prednosti hidroelektrana | Nedostaci hidroelektrana | Države s najvećom proizvodnjom hidro-električne energije | Deset najvećih hidroelektrana u svijetu | Hidroelektrane u Hrvatskoj | Izvori | Poveznice | Vanjske poveznice | Navigacijski izbornikTechnical Report, Version 2Zajedničkom poslužiteljuHidroelektranaHEP Proizvodnja d.o.o. - Hidroelektrane u Hrvatskoj

          WordPress Information needed

          Bosc Connection Yimello Approaching Angry The produce zaps the market. 구성 기록되다 변경...